LSAT and Law School Admissions Forum

Get expert LSAT preparation and law school admissions advice from PowerScore Test Preparation.

User avatar
 Dave Killoran
PowerScore Staff
  • PowerScore Staff
  • Posts: 5852
  • Joined: Mar 25, 2011
|
#43434
Complete Question Explanation
(The complete setup for this game can be found here: lsat/viewtopic.php?t=16399)

The correct answer choice is (A)

Similar to question #21, when T is selected, from the rules and inferences we know that N, K, and H cannot be selected, and therefore F, G, and L must be selected:
J03_Game_#4_#22_diagram 1.png
Answer choice (A) is therefore correct.

Another way of attacking this question is to eliminate answers that contain a variable that cannot be selected with T: use the rule that N and T cannot be selected together to eliminate answer choice (E); then use the inference that K and T cannot be selected together to eliminate answer choices (C) and (D); then use the inference that K and H cannot be selected together to eliminate answer choice (B).
You do not have the required permissions to view the files attached to this post.

Get the most out of your LSAT Prep Plus subscription.

Analyze and track your performance with our Testing and Analytics Package.